A complete orthonormal system ${e_i}^infty_{i=1}$ in $H$ is a basis in $H$












1














enter image description hereenter image description hereenter image description here



I'm studying about Hilbert space from a book of functional analysis and I just read this theorem (2.1.10) and its' proof.



I cannot understand why $(y-x)perp e_i$? why is it implied?










share|cite|improve this question






















  • $langle y,e_irangle = langle sum_j langle x,e_j rangle e_j, e_i rangle = sum_j langle x, e_j rangle langle e_j , e_i rangle = langle x, e_i rangle$
    – mathworker21
    yesterday
















1














enter image description hereenter image description hereenter image description here



I'm studying about Hilbert space from a book of functional analysis and I just read this theorem (2.1.10) and its' proof.



I cannot understand why $(y-x)perp e_i$? why is it implied?










share|cite|improve this question






















  • $langle y,e_irangle = langle sum_j langle x,e_j rangle e_j, e_i rangle = sum_j langle x, e_j rangle langle e_j , e_i rangle = langle x, e_i rangle$
    – mathworker21
    yesterday














1












1








1


1





enter image description hereenter image description hereenter image description here



I'm studying about Hilbert space from a book of functional analysis and I just read this theorem (2.1.10) and its' proof.



I cannot understand why $(y-x)perp e_i$? why is it implied?










share|cite|improve this question













enter image description hereenter image description hereenter image description here



I'm studying about Hilbert space from a book of functional analysis and I just read this theorem (2.1.10) and its' proof.



I cannot understand why $(y-x)perp e_i$? why is it implied?







functional-analysis hilbert-spaces normed-spaces orthonormal complete-spaces






share|cite|improve this question













share|cite|improve this question











share|cite|improve this question




share|cite|improve this question










asked yesterday









ChikChak

790418




790418












  • $langle y,e_irangle = langle sum_j langle x,e_j rangle e_j, e_i rangle = sum_j langle x, e_j rangle langle e_j , e_i rangle = langle x, e_i rangle$
    – mathworker21
    yesterday


















  • $langle y,e_irangle = langle sum_j langle x,e_j rangle e_j, e_i rangle = sum_j langle x, e_j rangle langle e_j , e_i rangle = langle x, e_i rangle$
    – mathworker21
    yesterday
















$langle y,e_irangle = langle sum_j langle x,e_j rangle e_j, e_i rangle = sum_j langle x, e_j rangle langle e_j , e_i rangle = langle x, e_i rangle$
– mathworker21
yesterday




$langle y,e_irangle = langle sum_j langle x,e_j rangle e_j, e_i rangle = sum_j langle x, e_j rangle langle e_j , e_i rangle = langle x, e_i rangle$
– mathworker21
yesterday










1 Answer
1






active

oldest

votes


















1














In Hilbert spaces the dot product is closed to infinite sums as well if the series converges. So because the system is orthonormal:



$langle y-x,e_jrangle=langle sum_{i=1}^inftylangle x,e_irangle e_i,e_jrangle-langle x,e_jrangle=sum_{i=1}^inftylangle x,e_iranglelangle e_i,e_jrangle-langle x,e_jrangle=$



$=langle x,e_jrangle-langle x,e_jrangle=0$






share|cite|improve this answer





















  • Could you explain why $langle sum_{i=1}^inftylangle x,e_irangle e_i,e_jrangle-langle x,e_jrangle=sum_{i=1}^inftylangle x,e_iranglelangle e_i,e_jrangle-langle x,e_jrangle$?
    – ChikChak
    yesterday












  • For each $N$ we have $langle sum_{i=1}^N langle x,e_irangle e_i,e_jrangle$=$sum_{i=1}^Nlangle x,e_iranglelangle e_i,e_jrangle$. Here I'm only using the definition of dot product. And now you can take $Ntoinfty$ on both sides using the continuity of dot product.
    – Mark
    yesterday












  • By the way, how is that book called?
    – Mark
    yesterday










  • Milman Eidelman Tsolomitis, Functional analysis An Introduction(T)
    – ChikChak
    17 hours ago











Your Answer





StackExchange.ifUsing("editor", function () {
return StackExchange.using("mathjaxEditing", function () {
StackExchange.MarkdownEditor.creationCallbacks.add(function (editor, postfix) {
StackExchange.mathjaxEditing.prepareWmdForMathJax(editor, postfix, [["$", "$"], ["\\(","\\)"]]);
});
});
}, "mathjax-editing");

StackExchange.ready(function() {
var channelOptions = {
tags: "".split(" "),
id: "69"
};
initTagRenderer("".split(" "), "".split(" "), channelOptions);

StackExchange.using("externalEditor", function() {
// Have to fire editor after snippets, if snippets enabled
if (StackExchange.settings.snippets.snippetsEnabled) {
StackExchange.using("snippets", function() {
createEditor();
});
}
else {
createEditor();
}
});

function createEditor() {
StackExchange.prepareEditor({
heartbeatType: 'answer',
autoActivateHeartbeat: false,
convertImagesToLinks: true,
noModals: true,
showLowRepImageUploadWarning: true,
reputationToPostImages: 10,
bindNavPrevention: true,
postfix: "",
imageUploader: {
brandingHtml: "Powered by u003ca class="icon-imgur-white" href="https://imgur.com/"u003eu003c/au003e",
contentPolicyHtml: "User contributions licensed under u003ca href="https://creativecommons.org/licenses/by-sa/3.0/"u003ecc by-sa 3.0 with attribution requiredu003c/au003e u003ca href="https://stackoverflow.com/legal/content-policy"u003e(content policy)u003c/au003e",
allowUrls: true
},
noCode: true, onDemand: true,
discardSelector: ".discard-answer"
,immediatelyShowMarkdownHelp:true
});


}
});














draft saved

draft discarded


















StackExchange.ready(
function () {
StackExchange.openid.initPostLogin('.new-post-login', 'https%3a%2f%2fmath.stackexchange.com%2fquestions%2f3062080%2fa-complete-orthonormal-system-e-i-infty-i-1-in-h-is-a-basis-in-h%23new-answer', 'question_page');
}
);

Post as a guest















Required, but never shown

























1 Answer
1






active

oldest

votes








1 Answer
1






active

oldest

votes









active

oldest

votes






active

oldest

votes









1














In Hilbert spaces the dot product is closed to infinite sums as well if the series converges. So because the system is orthonormal:



$langle y-x,e_jrangle=langle sum_{i=1}^inftylangle x,e_irangle e_i,e_jrangle-langle x,e_jrangle=sum_{i=1}^inftylangle x,e_iranglelangle e_i,e_jrangle-langle x,e_jrangle=$



$=langle x,e_jrangle-langle x,e_jrangle=0$






share|cite|improve this answer





















  • Could you explain why $langle sum_{i=1}^inftylangle x,e_irangle e_i,e_jrangle-langle x,e_jrangle=sum_{i=1}^inftylangle x,e_iranglelangle e_i,e_jrangle-langle x,e_jrangle$?
    – ChikChak
    yesterday












  • For each $N$ we have $langle sum_{i=1}^N langle x,e_irangle e_i,e_jrangle$=$sum_{i=1}^Nlangle x,e_iranglelangle e_i,e_jrangle$. Here I'm only using the definition of dot product. And now you can take $Ntoinfty$ on both sides using the continuity of dot product.
    – Mark
    yesterday












  • By the way, how is that book called?
    – Mark
    yesterday










  • Milman Eidelman Tsolomitis, Functional analysis An Introduction(T)
    – ChikChak
    17 hours ago
















1














In Hilbert spaces the dot product is closed to infinite sums as well if the series converges. So because the system is orthonormal:



$langle y-x,e_jrangle=langle sum_{i=1}^inftylangle x,e_irangle e_i,e_jrangle-langle x,e_jrangle=sum_{i=1}^inftylangle x,e_iranglelangle e_i,e_jrangle-langle x,e_jrangle=$



$=langle x,e_jrangle-langle x,e_jrangle=0$






share|cite|improve this answer





















  • Could you explain why $langle sum_{i=1}^inftylangle x,e_irangle e_i,e_jrangle-langle x,e_jrangle=sum_{i=1}^inftylangle x,e_iranglelangle e_i,e_jrangle-langle x,e_jrangle$?
    – ChikChak
    yesterday












  • For each $N$ we have $langle sum_{i=1}^N langle x,e_irangle e_i,e_jrangle$=$sum_{i=1}^Nlangle x,e_iranglelangle e_i,e_jrangle$. Here I'm only using the definition of dot product. And now you can take $Ntoinfty$ on both sides using the continuity of dot product.
    – Mark
    yesterday












  • By the way, how is that book called?
    – Mark
    yesterday










  • Milman Eidelman Tsolomitis, Functional analysis An Introduction(T)
    – ChikChak
    17 hours ago














1












1








1






In Hilbert spaces the dot product is closed to infinite sums as well if the series converges. So because the system is orthonormal:



$langle y-x,e_jrangle=langle sum_{i=1}^inftylangle x,e_irangle e_i,e_jrangle-langle x,e_jrangle=sum_{i=1}^inftylangle x,e_iranglelangle e_i,e_jrangle-langle x,e_jrangle=$



$=langle x,e_jrangle-langle x,e_jrangle=0$






share|cite|improve this answer












In Hilbert spaces the dot product is closed to infinite sums as well if the series converges. So because the system is orthonormal:



$langle y-x,e_jrangle=langle sum_{i=1}^inftylangle x,e_irangle e_i,e_jrangle-langle x,e_jrangle=sum_{i=1}^inftylangle x,e_iranglelangle e_i,e_jrangle-langle x,e_jrangle=$



$=langle x,e_jrangle-langle x,e_jrangle=0$







share|cite|improve this answer












share|cite|improve this answer



share|cite|improve this answer










answered yesterday









Mark

6,080415




6,080415












  • Could you explain why $langle sum_{i=1}^inftylangle x,e_irangle e_i,e_jrangle-langle x,e_jrangle=sum_{i=1}^inftylangle x,e_iranglelangle e_i,e_jrangle-langle x,e_jrangle$?
    – ChikChak
    yesterday












  • For each $N$ we have $langle sum_{i=1}^N langle x,e_irangle e_i,e_jrangle$=$sum_{i=1}^Nlangle x,e_iranglelangle e_i,e_jrangle$. Here I'm only using the definition of dot product. And now you can take $Ntoinfty$ on both sides using the continuity of dot product.
    – Mark
    yesterday












  • By the way, how is that book called?
    – Mark
    yesterday










  • Milman Eidelman Tsolomitis, Functional analysis An Introduction(T)
    – ChikChak
    17 hours ago


















  • Could you explain why $langle sum_{i=1}^inftylangle x,e_irangle e_i,e_jrangle-langle x,e_jrangle=sum_{i=1}^inftylangle x,e_iranglelangle e_i,e_jrangle-langle x,e_jrangle$?
    – ChikChak
    yesterday












  • For each $N$ we have $langle sum_{i=1}^N langle x,e_irangle e_i,e_jrangle$=$sum_{i=1}^Nlangle x,e_iranglelangle e_i,e_jrangle$. Here I'm only using the definition of dot product. And now you can take $Ntoinfty$ on both sides using the continuity of dot product.
    – Mark
    yesterday












  • By the way, how is that book called?
    – Mark
    yesterday










  • Milman Eidelman Tsolomitis, Functional analysis An Introduction(T)
    – ChikChak
    17 hours ago
















Could you explain why $langle sum_{i=1}^inftylangle x,e_irangle e_i,e_jrangle-langle x,e_jrangle=sum_{i=1}^inftylangle x,e_iranglelangle e_i,e_jrangle-langle x,e_jrangle$?
– ChikChak
yesterday






Could you explain why $langle sum_{i=1}^inftylangle x,e_irangle e_i,e_jrangle-langle x,e_jrangle=sum_{i=1}^inftylangle x,e_iranglelangle e_i,e_jrangle-langle x,e_jrangle$?
– ChikChak
yesterday














For each $N$ we have $langle sum_{i=1}^N langle x,e_irangle e_i,e_jrangle$=$sum_{i=1}^Nlangle x,e_iranglelangle e_i,e_jrangle$. Here I'm only using the definition of dot product. And now you can take $Ntoinfty$ on both sides using the continuity of dot product.
– Mark
yesterday






For each $N$ we have $langle sum_{i=1}^N langle x,e_irangle e_i,e_jrangle$=$sum_{i=1}^Nlangle x,e_iranglelangle e_i,e_jrangle$. Here I'm only using the definition of dot product. And now you can take $Ntoinfty$ on both sides using the continuity of dot product.
– Mark
yesterday














By the way, how is that book called?
– Mark
yesterday




By the way, how is that book called?
– Mark
yesterday












Milman Eidelman Tsolomitis, Functional analysis An Introduction(T)
– ChikChak
17 hours ago




Milman Eidelman Tsolomitis, Functional analysis An Introduction(T)
– ChikChak
17 hours ago


















draft saved

draft discarded




















































Thanks for contributing an answer to Mathematics Stack Exchange!


  • Please be sure to answer the question. Provide details and share your research!

But avoid



  • Asking for help, clarification, or responding to other answers.

  • Making statements based on opinion; back them up with references or personal experience.


Use MathJax to format equations. MathJax reference.


To learn more, see our tips on writing great answers.





Some of your past answers have not been well-received, and you're in danger of being blocked from answering.


Please pay close attention to the following guidance:


  • Please be sure to answer the question. Provide details and share your research!

But avoid



  • Asking for help, clarification, or responding to other answers.

  • Making statements based on opinion; back them up with references or personal experience.


To learn more, see our tips on writing great answers.




draft saved


draft discarded














StackExchange.ready(
function () {
StackExchange.openid.initPostLogin('.new-post-login', 'https%3a%2f%2fmath.stackexchange.com%2fquestions%2f3062080%2fa-complete-orthonormal-system-e-i-infty-i-1-in-h-is-a-basis-in-h%23new-answer', 'question_page');
}
);

Post as a guest















Required, but never shown





















































Required, but never shown














Required, but never shown












Required, but never shown







Required, but never shown

































Required, but never shown














Required, but never shown












Required, but never shown







Required, but never shown







Popular posts from this blog

Mario Kart Wii

What does “Dominus providebit” mean?

Antonio Litta Visconti Arese